إعـــــــلان

تقليص
لا يوجد إعلان حتى الآن.

الي معاه بكلوريس كيمياء حيوي .. ضروري ادخلووو

تقليص
X
 
  • تصفية - فلترة
  • الوقت
  • عرض
إلغاء تحديد الكل
مشاركات جديدة

  • الي معاه بكلوريس كيمياء حيوي .. ضروري ادخلووو

    السلام عليكم رحمة الله

    اخواني بسألكم ..

    انا خريجة كلية العلوم .. كيماء حيويه

    انا الي عارفتووو انوو التصنيفراح يكون فني

    بس طريقه الاختبار في كل الاقسام ولا فقط في التخصص

    لاني دخلت موقع الهيئه الصحيه ولقيت مكتوب اختبار خاص للكيميا الحيوي

    شوفو الرابط على اليمين


    http://arabic.scfhs.org.sa/new/pages...chnicians.html




    الله يسعدكم اذا فيه احد من كلية العلوم واختبر قريب يفيدنا كيف كان الاختبار

    الله يجزاكم الخير

  • #2
    مرحبا

    انا تخصصي احياء دقيقه - كليه علوم
    اختبرت بكل الاقسام والاختبار كان جدا رائع مافيه اي شي من اللي ينقال ذاكري تمام وتوكلي على الله
    بس خلي بالك خريجي كليه العلوم نفس الاختبار سواء كان احياء دقيقه او كيمياء حيوي والتركيز بالاختبار ع الاحياء الدقيقه
    ومثل ماقلتي التصنيف حايكون فنيه واذا توظفتي يتغير بعد سنتين لاخصائيه هذا الكلام تاكدي منه انا قبل شهرين مختبره يعني قفلي اذنك عن الكلام الفاضي لانه انا سمعت كثير كلام احبطني

    اتمنى اكون افدتك بشي والله يوفقك عزيزتي

    تعليق


    • #3
      هلا اختي انا تخصصي كيمياء حيوي وامتحنت الاسبوع الي فات والحمد الله نجحت بنسبة عاليه ولاتخافي ترا الامتحان يجي في كل لاقسام وهو نفس امتحان الفني
      انت ذاكري من الملخصات اللي في المنتدى وركزي على نماذج الامتحانات وتضمني النجاح ان شاء الله ^_^

      تعليق


      • #4
        أنــآ اختبرت كل اسبوعين والاسئلة بس كانت بالمايكروا

        ولم اوفق في الاختبار
        اللي اختبر وتخصصه أحيـاء دقيقة يقولـــــــــــي///لاني شوفت لي دكتورة تذاكرلي المنهج
        يمكن يكونوا غيروا الاسئلة

        تعليق


        • #5
          الحزن العتيق .. سكر زياده

          الله يسعدكم والله ريحتووووووني باجاباتكم

          اختباري باذن الله في شعبان ان شاء الله اتوفق وانجح

          الله يسعدكم يارب ويفرحكم بالوظيفه





          ورده تحت المطر

          ان شاء الله المره الجايه تتوفقي بالامتحان

          انتي طيب بأيت مدينة ؟؟

          تعليق


          • #6
            بنات بسألكم هل النتيجه تطلع لك على طول بنفس الوقت

            ومتى تقدري تستلمي بطاقه التصنيف

            وكمان بسالكم لمن قدمتوو طلبوا منكم دورة الانعاش القلبي الرئوي ؟؟

            وايش طلبوا منكم اوراق ؟؟

            تعليق


            • #7
              انا من المدينه
              والنتيجه تطلع ع طول/وبطاقه التصنيف تاخذينها بعد اسبوعين زي ماسمعت
              ولازم يطلبوها منك دورة الانعاش الرئوي
              الاوراق اللي طلبوها :
              صورة من وثيقة التخرج وصورة من دورة الانعاش ودفع رسوم عن طريق بنك الاهلي انتي لارحتي لهم يعطوكي رقم حساب البنك
              صورة من شهادة التدريب والتقيييم وصورة من كرت العايله او صورة من البطاقة الشخصية
              رسوم الهيئة 840 ورسوم الاختبار 350 ترى كل وحده
              لها حساب مختلف

              تعليق


              • #8
                ماشاء الله اخت وردة تحت المطر ماقصرت جاوبتك على اسئلتك
                انا لسه ماستلمت البطاقه وقالولي راجعي بعد اسبوعين وموعدي معاهم الاربعاء الجاي ^_^


                وردة تحت المطر لاتزعلي نفسك وانشاء الله بتنجحي في المرة الجايه
                حاولي تركزي على النماذج وترا الامتحان سهل مايبغاله انك تروحي لدكتورة وكذا
                كل الي تحتاجيه تعرفي ايش تذاكري وكيف وزي ماقلتلك النماااااذج مرة راح تسهل عليكي
                وموفقه ان شاء الله

                تعليق


                • #9
                  العفو اختي سكر ماسويت الا الواجب
                  ماعندي نماذج اسئلة فني احياء دقيقة والاسئلة اللي بالمنتدى تختلف عن الاسئلة فني أحياء دقيقة
                  القرار هذا جديد ماله غير شهرين
                  ودراستي كانت بالعربي مضطرة اشوف لي دكتورة تشرحلي

                  تعليق


                  • #10
                    شكرا على اجوبتك

                    طيب البنات قالوو ان الاسئله راح تكون في كل الاقسام

                    مو بس احياء ؟؟

                    تعليق


                    • #11
                      العفو خيتو
                      البنات.اللي قالوا لك كذا وش تخصصاتهم ومتى اختبروا ؟
                      اللي قريته من المنتدى واللي توهم اختبروا قالوا ان اسئلة فني الكيمياء الحيوية هي نفسها اسئلة فني مختبر

                      تعليق


                      • #12
                        ارجعي اقرأي اردود الاولى


                        اتوقع كاتبه مختبره من شهر ووحده اختبرت الاسبوع الي فات


                        احسن اتصلي على الهيئه بنفسك واتاكدي


                        الله يوفقنا

                        تعليق


                        • #13
                          برجع أتصل عليــــــــــــــــهم..
                          اللي اختبروا وجاتهم الاسئلة نفس اسئلة فني مختبر ونفس تخصصي اختبروا قبل شهرين ( قبل ما يجي القرار الجديد)

                          تعليق


                          • #14
                            HEMATOLOGY

                            1- Vitamin K antagonist :
                            a- warfarin
                            b- Heparin
                            c- Protein C
                            d- Antithrombin III

                            2- One of the intrinsic pathway
                            a- factor XI
                            b- factor XIII
                            c- factor I
                            d- factor VII

                            3- Para hemophilia is the deficiency of
                            a- factor VIII
                            b- factor IX
                            c- factor V
                            d- factor VII

                            4- Eosinophilia is seen in :
                            a- food sensitivity
                            b- Drug sensitivity
                            c- Atopic dermatitis
                            d- all of the above

                            5- Multiple myeloma is a neoplastic proliferation of:
                            a- lymphocytes
                            b- Granulocytes
                            c- Plasma cells
                            d- Monocytes

                            6- Test for intrinsic pathway:
                            a- bleeding time
                            b- Thrombin time
                            c- Prothrombin time
                            d- Partial thromboplastin time PTT


                            7- Paul-Bunnel test is done to diagnose:
                            a- multiple myeloma
                            b- Hodgkin’s disease
                            c- Infectious mononucleosis
                            d- all of the above

                            8- increased platelet count is :
                            a- thrombocytopenia
                            b- thrombopoietin
                            c- thrombocytosis
                            d- all of the above

                            9- Decreased platelet count is:
                            a- thrombocytopenia
                            b- Thrombopoietin
                            c- Thrombocytosis
                            d- all of the above

                            10- All these are causes of thrombocytopenia except:
                            a- cytotoxic drugs
                            b- Aplastic anemia
                            c- Hemorrhage
                            d- Radiotherapy

                            11- Prothrombin time is done to test:
                            a- Intrinsic pathway only
                            b- Extrinsic pathway only
                            c- Extrinsic and common pathways
                            d- Intrinsic and common pathways

                            12- Normal bleeding time by Duke’s method:
                            a- 2-7 minutes
                            b- 2-7 seconds
                            c- 2-4 minutes
                            d- 2-4 seconds

                            13- Normal partial thromboplastin time (PPT) is :
                            a- 3-4 minutes
                            b- 30-45 seconds
                            c- 12-15 seconds
                            d- 12-15 minutes

                            14- Hemophilia A is the deficiency of :
                            a- factor V
                            b- factor VIII
                            c- factor IX
                            d- all of the above

                            15-the most common form of leukemia in children is:
                            a- acute lymphoblastic leukemia
                            b- Chronic lymphocytic leukemia
                            c- Acute myeloid leukemia
                            d- Chronic myeloid leukemia

                            16- Bence-Jones protein is present in cases of:
                            a- chronic myeloid myeloma
                            b-acute myeloid myeloma
                            c- Hodgkin’s lymphoma
                            d- multiple myeloma

                            17- Reed-Sternberg cells are found in cases of :
                            a- acute lymphoblastic leukemia
                            b- Non Hodgkin’s lymphoma
                            c- Hodgkin’s lymphoma
                            d- Multiple myeloma

                            18- Normal platelet count is :
                            a- 150,000 to 450,000/min3
                            b- 400,000 to 800,000/min3
                            c- 4,000 to 11,000 /min3
                            d- 50,000 to 100,000/min3

                            19- Antithrombin III inhibits:
                            a- factor Va
                            b- factor VIIIa
                            c- factor Xa
                            d- all of the above

                            20- Heparin potentiate the action of :
                            a- protein C
                            b- protein S
                            c- antithrombin III
                            d- warfarin


                            21- Factor II of blood clotting is:
                            a- Christmas factor
                            b- Fibrinogen
                            c- Prothrombin
                            d- Thromboplastin

                            22- One of fibrinogen group is : I V VIII XIII
                            a- II
                            b- V
                            c- VII
                            d- IX

                            23- Fibrinogen is converted to soluble fibrin by:
                            a- prothrombin
                            b- Thromboplastin
                            c- Thrombin
                            d- all of the above

                            24- Thrombopoitin control the formation of:
                            a- red blood cells
                            b- White blood cells
                            c- platelets
                            d- non of the above

                            25- Normal prothrombin time (PT) is:
                            a- 30-45 seconds
                            b- 30-45 minutes
                            c- 12-15 seconds
                            d- 12-15 minutes

                            26- Parasitic disease is associated with:
                            a- monocytosis - bacteria
                            b- Lymphocytosis - virus
                            c- Basophilia - sensitive
                            d- Eosinophilia

                            27- Philadelphia chromosome is diagnostic for:
                            a- acute lymphoblastic leukemia
                            b- Acute myeloid leukemia
                            c- Chronic lymphocytic leukemia
                            d- chronic myeloid leukemia
                            28- Normal fibrinogen level:
                            a- 150-400 gm%
                            b- 150-400 mg %
                            c- 15-40 mg %
                            d- 15-40 gm %

                            29-infectious mononucleosis is caused by:
                            a- echo virus
                            b- coxsaki virus
                            c- Epstein Barr virus EBV
                            d- Cytomegalo virus

                            30- Atypical lymphocytosis is seen in cases of:
                            a- Hodgkin’s lymphoma
                            b- Multiple myeloma
                            c- Infectious mononucleosis
                            d- Chronic lymphocytic leukemia

                            31-monospot test is done to diagnose:
                            a- Acute myeloid leukemia
                            b- Acute lymphoblastic leukemia
                            c- Infectious mononucleosis
                            d- Infectious lymphocytosis

                            32- The test which depend on blood platelets & capillary fragility is:
                            a- prothrombin time
                            b- Thrombin time
                            c- Bleeding time
                            d- Clotting time

                            33- Fibrin is broken to fibrin degradation products by the action of:
                            a- Prothrombin
                            b- Thrombin
                            c- Plasminogen
                            d- Plasmin





                            34- Acute myeloid leukemia is characterized by:
                            a- low neutrophil alkaline phosphatase
                            b- Myeloblast with Auer rods
                            c- Neutrophil with Pleger-Huet anomaly
                            d- all of the above

                            35- Plasminogen is converted to plasmin by :
                            a- Heparin
                            b-Histamine
                            c- Urokinase
                            d- Serotonin

                            36- Increase D-dimers and fibrin degradation products are seen in cases of:
                            a- Hemophilia A
                            b- Vitamin K deficiency
                            c- Diffuse intravascular coagulation
                            d- Von Willebrand disease

                            37- Activated protein C degrades:
                            a- factor IXa
                            b- Factor VIIIa
                            c- Factor Xa
                            d- Factor Xia

                            38- Heparin is found in
                            a- Neutrophil
                            b- Basophil
                            c- Acidophil
                            d- all of the above

                            39- In hemophilia A the test which will be prolonged is
                            a- PT
                            b- PTT
                            c- Bleeding time
                            d- all of the above

                            40- Bleeding due to overdose of heparin is managed by giving:
                            a- Vit K
                            b- Vit C
                            c- Vit A
                            d- Protamin sulphate
                            41- Streptokinase and staphylokinase convert:
                            a- Prothrombin to thrombin
                            b- Fibrinogen to fibrin
                            c- Soluble fibrin to insoluble fibrin
                            d- Plasminogen to plasmin

                            42- Test for platelet function:
                            a- Clot retraction
                            b- Platelet aggregation
                            c- Platelet adhesion
                            d- all of the above

                            43- Prolonged PT occurs in cases of deficiency of:
                            a- Factor III
                            b- Factor IV
                            c- Factor V
                            d- all of the above

                            44- normal thrombin time (TT):
                            a- 30-45 sec
                            b- 2-4 min
                            c- 3-9 min
                            d- 10-20 sec

                            45- cause of vitamin K deficiency:
                            a- Obstructive jaundice
                            b- Prolonged use of antibiotics
                            c- Inadequate intake
                            d- all of the above

                            46- Cause of Hyper- Coagulable state:
                            a- Aplastic anemia
                            b- Cytotxic drugs
                            c- Polcythemia
                            d- Radiotherapy

                            47-Physiological cause of neutrophilia:
                            a- New born
                            b- Radiotherapy
                            c- Cytotoxic drugs
                            d- Prolonged use of antibiotics
                            48- Leucocytosis characterized by the presence of immature cells and high neutrophil alkaline phosphatase:
                            a- chronic myeloid leukemia
                            b- Acute myeloid leukemia
                            c- Leukaemid reaction
                            d- non of the above

                            49- Normal coagulation time (CT):
                            a- 3-9 min
                            b- 3-9 sec
                            c- 30-40 sec
                            d- 30-40 min

                            50- The test which measures the clotting time of citrated plasma accelerated by the addition of a clotting factor activator (kaolin) , phospholipids and calcium:
                            a- coagulation time
                            b- Prothrombin time
                            c- Partial thromboplastin time
                            d- Thrombin time

                            51- The test which measures the clotting time of citrated plasma to which thromboplastin and calcium has been added:
                            a- thrombin time
                            b- Prothrombin time
                            c- Coagulation time
                            d- Partial thromboplastin time

                            52- The test which is widely used as a control and follow up test to control anticoagulant treatment:
                            a- APTT
                            b- PTT
                            c- PT
                            d- TT

                            53- A disease characterized by progressive neoplastic proliferation of immature white cell precursor:
                            a- acute leukemia
                            b- Chronic leukemia
                            c- Lymphoma
                            d- Multiple myeloma


                            54- The absolute lymphocyte count may be up to 300,000 or more between 70 and 90 % of white cells in the blood film appear as small lymphocytes . THE CASE IS:
                            a- Acute myeloid leukemia
                            b- Acute lymphoblastic leukemia
                            c- Chronic myeloid leukemia
                            d- Chronic lymphocytic leukemia

                            55- Variation in red cells size:
                            a- Poikilcytosis
                            b- Anisocytosis
                            c- Reticulocytosis
                            d- Leukocytosis

                            56- Dark red cells with no area of central pallor:
                            a- Stomatocyte
                            b- Spherocyte
                            c- Acathocyte
                            d- Schistocyte

                            57- Microcytic hypochromic anemia
                            a- hereditary spherocytosis
                            b- Sickle cell anemia
                            c- Iron deficiency anemia
                            d- Vit B12 deficiency anemia

                            58- Target cells are seen in cases of:
                            a- folic acid deficiency
                            b- Iron deficie.ncy anemia
                            c- Vit B12 deficiency anemia
                            d- Thalassemia

                            59- Red cells with elongated area of central pallor:
                            a- spherocyte
                            b- Schistocyte
                            c- Stomatocyte
                            d- Elliptocutes



                            60- ……………….. Symmetric, short , sharp projection from the red cells and seen in iron deficiency anemia:
                            a- echinocyte
                            b- Acanthocyte
                            c- Elliptocyte
                            d- Ovalocyte

                            61- ……………… is a condition in which the absorption of vit B12 is greatly impaired due to failure or marked reduction of intrinsic factor secretion:
                            a- fauvism
                            b- fanconi’s anemia
                            c-sickle cell anemia
                            d- thalassemia
                            e- pernicious anemia

                            62- Hyperchromic cells are seen in:
                            a- iron deficiency anemia
                            b- Thalassemia
                            c- Hereditary spherocytosis
                            d- Sickle cell anemia

                            63- A prolonged low rate of bleeding results in:
                            a- normochromic anemia
                            b- Hypochromic anemia
                            c- Hyperchromic anemia
                            d- non of the above

                            64- Schilling test is done in diagnosis of:
                            a- iron deficiency anemia
                            b- Pernicious anemia
                            c- Aplastic anemia
                            d- folic acid deficiency


                            65- Defective synthesis of either alpha or beta chains of normal hemoglobin
                            cause:
                            a- sickle cell anemia
                            b- Aplastic anemia
                            c- Pernicious anemia
                            d- Thalassemia

                            66- Neutrophils represent…………… of circulating leukocyte:
                            a- 2-8 %
                            b- 0-1 %
                            c- 50-70 %
                            d- 2-4 %

                            67- …………. are non nucleated, biconcave shaped cells:
                            a- platelet
                            b- Leukocyte
                            c- Erythrocyte
                            d- Macrophages

                            68- …………….. represent 50-70 % of total leukocytes
                            a- lymphocytes
                            b- Neutrophils
                            c- Monocytes
                            d- Eosinophilis

                            69- …………… have a characteristic biffed nucleus and their cytoplasm is filled with large refractile granules that stain red in blood smear
                            a- neutrophils
                            b- Eosinophilis
                            c- Basophiles
                            d- Lymphocytes

                            70- The cell which is responsible for antibody production is:
                            a- moncytes
                            b- T-lymphocytes
                            c- B-lymphocytes = plasma cell
                            d- Neutrophils


                            71- …… is a curved cell with sharp ends seen in haemoglobinopathies (HBS)
                            a- sickle cell
                            b- Spherocyte
                            c- Ovalocyte
                            d- Stomatocyte


                            72- All of the following is correct about sickle cell anemia except:
                            a- leg ulcers
                            b- Gall stones
                            c- Enlargement of spleen
                            d- Attacks of pain

                            73- ……….. is a single, large, rounded , dark , purple remnant of nucleus
                            a- Heinz body
                            b- Howeel-Jolly body
                            c- Pappenheimer body
                            d- Cabot ring

                            74- Agranulocyte:
                            a- neutrophil
                            b- Lymphocyte
                            c- Basophile
                            d- Eosinophil

                            75- Pica ( craving to eat unusual substance such as clay or ice) is one of the symptoms of:
                            a- G6PD deficiency
                            b- Thalassemia
                            c- Megaloblastic anemia
                            d- Iron deficiency anemia

                            76- In …………….. there’s a decreased or absent hemosiderin in bone marrow
                            a- sideroblastic anemia
                            b- Iron deficiency anemia
                            c- Megaloblastic anemia
                            d- Hemolytic anemia

                            77- Chloramphenicol may cause …………. anemia in long term therapy
                            a- iron deficiency
                            b- Vit B12 deficiency
                            c- folic acid deficiency
                            d- Aplastic anemia

                            78- ………. is the fluid (with anticoagulant) component of blood , it contains salt & organic compounds:
                            a- plasma
                            b- Serum
                            c- Hemoglobin
                            d- Billirubin

                            79- Poikilocytosis is:
                            a- variation in red cell size
                            b- Variation in red cell color
                            c- Variation in red cell shape
                            d- non of the above

                            80- Red cell fragments:
                            a- echinocyte
                            b- Elliptocyte
                            c- Schistocyte
                            d- Stomatocyte

                            81- It is a defect of red cell member
                            a- Thalassemia
                            b- Sickle cell anemia
                            c- Hereditary spherocytosis
                            d- Megaloblastic anemia

                            82- All of the following is correct regarding spherocytosis except:
                            a- normocytic normochromic anemia
                            b- Decreased reticulocyte count
                            c- Raised plasma bilirubin
                            d- Increased osmotic fragility

                            83- Heinz bodies are seen in cases of
                            a- hereditary spherocytosis
                            b- Hereditary elliplocytosis
                            c- G6PD deficiency
                            d- sickle cell anemia



                            84- ……………… is caused by substitution of amino acid (valine) instead of glutamic acid at position No.#6 in the beta chain of hemoglobin
                            a- Hb-A
                            b- Hb-A2
                            c- Hb –F
                            d- Hb –S sickle cell

                            85- Atrophy of the spleen is seen in cases of:
                            a- Thalassemia
                            b- Sickle cell anemia
                            c- G6PD deficiency
                            d- Hereditary elliplocytosis

                            86- iron deficiency lead to :
                            a- normocytic normochromic anemia
                            b- microcytic hypochromic anemia
                            c- macrocytic anemia
                            d- hemolytic anemia

                            87- Neurological symptoms are seen in cases of:
                            a- iron deficiency anemia
                            b- folic acid deficiency
                            c- Vit B12 deficiency
                            d- all of the above

                            88- Megaloplastic hematopoiesis is seen in cases of:
                            a- iron deficiency anemia
                            b- folic acid deficiency
                            c- Vit B12 deficiency
                            d- Vit C deficiency


                            89- Fanconi’s anemia is a type of :
                            a - vit B12 deficiency
                            b- aplastic anemia
                            c- Thalassemia
                            d- folic acid deficiency anemia




                            90- the most abundant leukocyte in a normal blood smear of adult is :
                            a- lymphocyte
                            b- Monocyte
                            c- Eosinophil
                            d- Neutrophil

                            91- The first line of defense against parasites:
                            a- neutrophils
                            b- Basophile
                            c- Eosinophil
                            d- Lymphocyte

                            92- …………….. play a role in immediate and delayed hypersensitivity:
                            a- monocyte
                            b- Lymphocyte
                            c- Eosinophil
                            d- Basophile

                            93- The largest leukocyte is :
                            a- neutrophils
                            b- Lymphocyte
                            c- Monocyte
                            d- Basophile

                            94- Cell which participate in cell mediated immunity:
                            a- monocyte
                            b- B- lymphocyte
                            c- T- lymphocyte
                            d- neutrophils

                            95- ……… promotes blood clotting and help to prevent blood loss from damaged blood vessels:
                            a- platelets
                            b- WBCs
                            c- RBCs
                            d- all of the above


                            96-antibody induced hemolytic disease in new born that is caused by blood group incompatibility between mother and fetus:
                            a- hemolytic uremic syndrome
                            b- Erythroblastosis fetalis
                            c- Hereditary spherocytosis
                            d- Thromboloc thrombocytopenic purpurea

                            97- Young red blood cell with cytoplasmic RNA:
                            a- spherocyte
                            b- Reticulocyte
                            c- Stomatocyte
                            d- elliptocyte

                            98- Normal adult hemoglobin tetramer is:
                            a- 2 alpha : 2 gama
                            b- 2 alpha : 2 beta
                            c- 2 alpha : 2 delta
                            d- 2 beta : 2 gama

                            99- …………… represent 2-4 % of total leukocyte:
                            a- neutrophils
                            b- Basophile
                            c- Eosinophil
                            d- Monocyte

                            100- ………… are small cytoplasmic fragment derived from megakaryocytic:
                            a- RBCs
                            b- WBCs
                            c- Platelet
                            d- non of the above

                            101 - ………….. is the reduction in the amount of circulating hemoglobin , red blood cells or both:
                            a- polycythemia
                            b- Anemia
                            c- Hemophilia
                            d- Leucopenia



                            102- Thalassemia is :
                            a- microcytic anemia
                            b- Macrocytic anemia
                            c- Normocytic anemia
                            d- non of the above

                            103- Vit B12 deficiency lead to :
                            a- hemolytic anemia
                            b- Microcytic anemia
                            c- Normocytic anemia
                            d- Megaloblastic anemia

                            104- Lymphocyte represent ………….. of total leukocyte:
                            a- 20-40 %
                            b- 50-70 %
                            c- 2-8 %
                            d- 1-5 %

                            105- Plumer- Vinson syndrome may be seen in cases of:
                            a- iron deficiency anemia
                            b- Vit B12 deficiency anemia
                            c- Aplastic anemia
                            d- folic acid deficiency anemia

                            106- peripheral, pale inclusions that push out the cell membrane and composed of hemoglobin:
                            a- cabot ring
                            b- Pappenheimer body
                            c- Howell-Jolly body
                            d- Heinz body

                            107- it’s an acute hemolytic anemia occurring after the ingestion of broad bean in individual with deficiency of G6PD :
                            a-thalassemia
                            b- Favism
                            c- Fanconi’s anemia
                            d- Cooley’s anemia




                            108-………………. is a multiple small , peripheral grape like purple clusters of iron:
                            a- cabot ring
                            b- Heinz body
                            c- Howell-Jolly body
                            d- Pappenheimer body

                            109- the blood smear gives the physician information concerning:
                            a- morphology of RBCs and platelet
                            b- Presence of abnormal inclusion
                            c- Presence of immature cells
                            d- all of the above

                            110- hypersplenism is one of the causes of :
                            a- iron deficiency anemia
                            b- Hemolytic anemia
                            c- Aplastic anemia
                            d- Megaloblastic anemia

                            111- Increased reticulocytes count is seen in cases of:
                            a- hereditary spherocytosis
                            b- G6PD deficiency
                            c- Sickle cell anemia
                            d- all of the above

                            112- The antibody which can pass the placenta:
                            a- Ig M
                            b- Ig G
                            c- Ig D
                            d- Ig E


                            113- ……… is an autoimmune disease in which there is an immune destruction of the acid and pepsin secreating cells of the stomach:
                            a- fanconi’s anemia
                            b- cooley’s anemia
                            c-pernicious anemia
                            d- non of the above

                            114- All of these are laboratory features of aplastic anemia except:
                            a- pancytopenia
                            b-markedly hypocellular marrow
                            c- Increased reticulocyte count
                            d- Markedly increase in serum erythropoietin

                            115- Secondary granules of neutrophils contain:
                            a- elastase
                            b- Myeloperoxidase
                            c- Lysozyme
                            d- Histamine

                            116- Monocytes represent ………………. of total leukocyte:
                            a- 0-1 %
                            b- 2-4 %
                            c- 2-8 %
                            d- 20-4 %

                            117- the reagent used for leukocyte count is :
                            a- citric acid
                            b- Acetic acid
                            c- Hydrochloric acid
                            d- Sulphoric acid









                            BACTERIOLOGY

                            118- All are Prokaryotic cells except:
                            a- Fungi
                            b- Bacteria
                            c - Chlamydia
                            d- Mycoplasma

                            119- Viruses:
                            a- Contain only DNA or RNA
                            b - They Contain ribosome
                            c- Did not affected by antibiotics
                            d- a+c

                            120- All of these are essential structure except:
                            a- Nuclear body
                            b- Spores
                            c- Cell wall
                            d- Plasma Membrane

                            121- ________ is giving the shape to the bacteria
                            a.- Cytoplasmic Membrane
                            b- Capsule
                            c- Cell Wall
                            d- All of the above

                            122- One of its functions is selective permeability
                            a- Cell wall
                            b- Plasma membrane
                            c- Capsule
                            d- Spores

                            123- They are responsible for Haemagglutination Phenomenon
                            a- Flagella
                            b- Fimbria
                            c- Capsule
                            d- Cell wall






                            124- Clostridium Tetani is:
                            a- Atrichous bacteria
                            b- Mono-trichous bacteria
                            c- Amphi-trichous bacteria
                            d- Peri-trichous bacteria

                            125- Short curved or straight rods, motile by single polar flagellum
                            a- spirochaeta
                            b- Vibrio
                            c- Escherichia
                            d- Lactobacillus

                            126- Small gram negative cocci, occur in pairs
                            a- staphylococcus
                            b- streptococcus
                            c- neisseria
                            d- non of the above

                            127- Transfer of genetic information from one bacterium to another by
                            bacteriophages is:
                            a. Transformation
                            b. Tansduction
                            c. Conjugation
                            d. Mutation

                            128- Salmonella are:
                            a- Obligatory Aerobic bacteria
                            b.- Obligatory Anaerobic bacteria
                            c- Facultative Anaerobic bacteria (H2S +)
                            d- Micro-aerophilic bacteria

                            129- According to pH, vibrio cholera is
                            a- Osmophilic bacteria
                            b- Basophilic bacteria
                            c- Acidophilic bacteria
                            d- Neutrophilic bacteria


                            130- Staphylococci are:
                            a- Atrichous bacteria
                            b- Mono-trichous bacteria
                            c- Amphi-trichous bacteria
                            d- Peri-trichous bacteria

                            131- During replication of DNA, copying errors may occur and this is called
                            a- Conjugation
                            b- Transduction
                            c- Transformation
                            d- Mutation

                            132- Obligatory Anaerobic bacteria
                            a- grow only in presence of oxygen
                            b- grow only in absence of oxygen
                            c- grow either in presence or absence of oxygen
                            d- grow in presence of oxygen traces and 5 – 10% CO2

                            133- Neutrophilic bacteria grow well at
                            a- pH 8.5 – 9.0
                            b- pH 7.2 – 7.4
                            c- pH 5.0 – 5.5
                            d- pH 2.5 – 3.0

                            134- The rate of cell death increase and bacterial growth stopped, this is
                            a- Adaptation phase
                            b- Exponential phase .
                            c- Stationary phase.
                            d- Decline Phase

                            135- Beta-hemolytic
                            a- Cause complete hemolysis of RBC’s
                            b- Cause chemical change of Hemoglobin in RBC’s
                            c- Do not cause hemolysis
                            d- None of them





                            146- Mesophilic bacteria grow at:
                            a- 37°C
                            b- 14°C
                            c- 60°C
                            d- 120°C

                            137- Circulation of Bacteria and its toxins in blood
                            a- Pyaemia
                            b- Toxemia
                            c.- Bacteremia - without toxins
                            d- Septicemia

                            138- Disinfections that applied on living or injured tissues:
                            a- Sterilization
                            b- Antiseptic مطهر
                            c- Sanitation
                            d- Decontamination

                            139- Hot air oven is used for sterilization of
                            a- Glass
                            b- Rubber Gloves
                            c.- Plastic Syringes
                            d- Catheters

                            140- Disinfectant for superficial fungal infection
                            a- Phenol
                            b- Potassium permanganate
                            c- Chlorine
                            d- Hypochlorite compounds

                            141- Rifampin
                            a- inhibit cell wall synthesis
                            b- inhibit protein synthesis
                            c- inhibit folic acid pathway
                            d- inhibit mRNA synthesis

                            142- Transacetylase inactivate
                            a- aminoglycosides
                            b- chloramphenicol
                            c- penicillin
                            d- cephalosporins

                            143- The color of gram positive bacteria is
                            a- Yellow
                            b- Black.
                            c- Pink
                            d- Violet

                            144- Selective media for fungi
                            a- blood agar
                            b- Mac Conkey agar
                            c- Nutrient agar
                            d- Sabourand’s dextrose agar

                            145- Histoplasma is a :
                            a- Systemic mycosis
                            b- Sub – Cutaneous mycosis
                            c- Cutaneous mycosis
                            d- Superficial mycosis

                            146- All are asexual spores produced by mould except
                            a- Conidio – spores
                            b- Sporangio – spores
                            c- Endospores
                            d- Arthro – spores

                            147- They reproduce only by Asexual reproduction
                            a- Blastomycosis
                            b- Deutromycosis
                            c- Ascomycetes
                            d- Zygomycetes

                            148- The functions of cell wall is all of the following except:
                            a- Giving the shape to the bacteria
                            b- Carrying somatic antigen
                            c-Selective permeability& transport of solutes = plasma membrane
                            d- Protect the bacteria from plasmolysis

                            149- Atrichous Bacteria are:
                            a- Bacteria contain one flagellum
                            b- Bacteria contain 2 flagella
                            c- Bacteria without flagella
                            d- Bacteria with a tuft of flagella

                            150-__________ are essential for host cell attachment:
                            a- Flagella
                            b- Fimbria
                            c- Spores
                            d- Capsules

                            151- Irregular clusters of spherical cells:
                            a- Streptococcus
                            b- Staphylococcus
                            c- Lactobacillus
                            d- Escherichia

                            152- Clostridium Botulinum is:
                            a- Obligatory Aerobic Bacteria
                            b- Facultative Anaerobic Bacteria
                            c- Obligatory Anaerobic Bacteria
                            d- Micro- aerophilic Bacteria

                            153- Basophilic Bacteria grow well at:
                            a- pH 8.5 – 9.0
                            b- pH 7.2 – 7.4
                            c- pH 5.0 – 5.5
                            d- None of the above

                            154- Cells are divided at high & constant rate:
                            a- Decline Phase
                            b- Stationary Phase
                            c- Log Exponential Phase
                            d- Adaptation Phase

                            155- The dominant bacterial species in dental plaque are:
                            a- Coagulase Negative Staphylococci
                            b- Lactobacillus
                            c- Bacteroides
                            d- Streptococcus Sanguis

                            156- The spread of Pyogenic Bacteria in blood stream to different organs & produce multiple abscess is:
                            a- Septicemia
                            b- Bacteremia
                            c-Toxemia
                            d- Pyaemia
                            157- Inhibit the growth of micro organisms
                            a- Bacteriostatic
                            b- Bactericidal
                            c- Fungicidal
                            d- Germicidal

                            158- To sterilize fluid damaged by heat:
                            a- Gaseous Sterilization
                            b- Heat Sterilization
                            c- Filtration
                            d- Ionizing Radiation

                            159- For water disinfection we use:
                            a- Hydrogen peroxide
                            b- Formaldehyde
                            c- Chlorine
                            d- Hypochlorite compounds

                            160- Mechanism of action of penicillin:
                            a- Block peptidoglycan synthesis
                            b- Inhibit peptidyglycan cross – linking
                            c- inhibit folic acid pathway
                            d- inhibit protein synthesis

                            161- Sulfonamides:
                            a- inhibit cell wall synthesis
                            b- inhibit protein synthesis
                            c- inhibit DNA synthesis
                            d- inhibit folic acid pathway

                            162- Acetylase inactivates:
                            a- B – Lactam antibiotics
                            b- Aminoglycosides
                            c- Cloramphenicol
                            d- All of the above

                            163- Ringworm disease is caused by
                            a- Zygomycetes
                            b- Ascomycetes
                            c- Blastomycosis
                            d- None of the above

                            164- For wet – mount technique we add:
                            a- NaoH
                            b- K oH
                            c- H2 O2
                            d- All of the above

                            165- They are transmitted by arthropods
                            a- Chlamydia
                            b- Spirochetes
                            c- Mycoplasma
                            d- All of the above

                            166- In the past they were listed as large viruses
                            a- Richettsia
                            b- Mycoplasma
                            c- Chlamydia
                            d- None of the above

                            167- Bacteria multiply by:
                            a- Replication cycle
                            b- Simple binary fission
                            c- Sexual reproduction
                            d- All of the above

                            168- It protects bacteria from antibiotics
                            a- Capsule
                            b- Cell membrane
                            c- Flagella
                            d- Fimbria

                            169- Vibro cholera is:
                            a- Mono –trichous bacteria
                            b- Atrichous bacteria
                            c- Lopho-trichous bacteria
                            d- Peri-trichous bacteria

                            170- Short rods, motile by peritrichous flagella
                            a- Spirochaeta
                            b- Lactobacillus
                            c- Escherichia coli
                            d- Vibrio
                            171- To take up soluble DNA fragments derived from other, closely related species is:
                            a. Mutation
                            b. Transformation
                            c. Transduction
                            d. Conjugation

                            172- Tuberculosis are
                            a- micro-airophilic
                            b- Facultative anaerobic
                            c- Obligatory anaerobic
                            d- Obligatory aerobic

                            173- According to pH, Lactobacillus is
                            a- Neutrophlic bacteria
                            b- Acidophilic bacteria
                            c- Basophilic bacteria
                            d- None of the above

                            174- Bacteria without cell Wall
                            a- Chlamydia
                            b- Rickettsia
                            c- Mycoplasma
                            d.- Spirochetes

                            175- Brucella Melitensis is
                            a- Obligatory aerobic bacteria
                            b- Obligatory anaerobic bacteria
                            c- Facultative anaerobic bacteria
                            d- Micro-aerophilic bacteria

                            176- Pseudomonas aeroginosa is
                            a- Peri-trichous bacteria
                            b- Lopho-trichous bacteria
                            c- Amphi-trichous bacteria
                            d- Monotrichous bacteria

                            177- Genetic information of bacteria is carried on
                            a- Messenger RNA
                            b- Transfer RN|A
                            c- Transcript RNA
                            d- Double – Stranded DNA
                            178- Thermophilic bacteria grow at
                            a- 60 – 80 °C
                            b- 0 - 20°C
                            c-. 37°C
                            d- 100 - 120°C

                            179- Acidophilic bacteria grow at
                            a- pH 7.2 – 7.4
                            b- pH 5.0 – 5.5
                            c- pH 8.5 – 9.0
                            d- None of the above

                            180- Mycoplasma is
                            a- Neutrophilic bacteria
                            b- Acidophilic bacteria
                            c- Basophilic bacteria
                            d- All of the above

                            181- It is the adaptation of bacteria to the fresh medium
                            a- Lag phase
                            b- Decline phase
                            c- Logarithmic Phase
                            d- Stationary phase

                            182- Bacteria which do not cause hemolysis is
                            a- Beta-Hemolytic
                            b- Alpha-Hemolytic
                            c- Gama Hemolytic
                            d- None of the above

                            183- Normal flora of Lower intestine are all of the following except:
                            a- Staphylococci
                            b- Diphtheroids
                            c- Shigella
                            d- Lactobacillus

                            184- Opportunistic pathogens are all of the following except:
                            a- Cause a disease when the host defense are suppressed.
                            b- Are normal flora of healthy body
                            c- Are greatly harmful
                            d- Do not invade the body or tissue.
                            185- For disinfection of mattresses :
                            a- Hot air oven
                            b- Autoclave
                            c- Ethylene Oxide
                            d- Hydrogen Peroxide

                            186- Pyschrophilic bacteria grow at:
                            a- 10٠°C
                            b- 6٠°C
                            c- 14°C
                            d- 37°C

                            187- Bacteria which contain chlorophyll
                            a- Heterotrophic bacteria
                            b- Autotrophic bacteria
                            c- Photosynthetic bacteria
                            d- All of the above

                            188- Tricophyton is one of
                            a- Yeast
                            b- Moulds
                            c- Dermatophyte
                            d- Dimorphic Fungi

                            189- Plastomyces is one of
                            a- Dermatophytes
                            b- Dimorphic Fungi
                            c- Yeast
                            d- Moulds

                            190- Color of gram negative bacteria is
                            a- Violet
                            b- Green
                            c- Red
                            d- Black

                            191- Acid Fast Bacteria
                            a- Salmonella
                            b- Shigella
                            c- M. Tuberculosis
                            d- E – Coli
                            192- Spherical or avoid cells occurring in chains
                            a- Staphylococci
                            b- Streptococci
                            c- Lactobacillus
                            d- Spiro chaeta

                            193- ……………… carries the genetic information
                            a- the envelope
                            b- the capsid
                            c- the nucleic acid
                            d- the prion

                            194- …………………… may be seen under light microscope
                            a- rota virus
                            b- influenza virus
                            c- herps virus
                            d- pox virus

                            195- viruses may be:
                            a- monomorphic
                            b- pleomorphic
                            c- dimorphic
                            d- all of the above

                            196- viral capside is formed of:
                            a- protein
                            b- glycogen
                            c- lipoprotein bilayer
                            d- glycoprotein

                            197- class III in Baltimor classification is:
                            a- double stranded DNA viruses
                            b- single stranded DNA viruses
                            c- double stranded RNA viruses
                            d- single stranded RNA viruses

                            198- Hierarchial virus classification system use the following characters except:
                            a- nature of nucleic acid
                            b- capside symmetry
                            c- diameter of viron & capside
                            d- virus molecular weight
                            199- in viral replication which is true:
                            a- penetration is the 1st step
                            b- assembly is the last step
                            c- relaease is the last step
                            d- all of the above

                            200- viron:
                            a- may be extracellular phase of virus
                            b- may be intracellular phase of virus
                            c- can grow and replicate
                            d- means “ virus – like “

                            201- pleomorphic viruses means :
                            a- virus which have constant shape
                            b- virus that may appear in 2 forms
                            c- virus that have not a constant morphology
                            d- virus that have spherical shape

                            202- vapor of gold is used in :
                            a- shadow casting technique
                            b- negative staining technique
                            c- positive staining technique
                            d- non of the above

                            203- direct diagnosis of virus :
                            a- ELISA Antibody
                            b- CFT
                            c- IFT
                            d- PCR virus

                            204- all of the following are required in cell culture except:
                            a- neutral PH
                            b- presence of buffer salts
                            c- presence of antibiotics
                            d- incubation at 20 C (( 30-37 C))

                            205- all of the following are diagnostic molecular biological technique except:
                            a- PCR
                            b- ELISA
                            c- nucleic acid hyperdization
                            d- DNA finger printing
                            206- PCR require all of the following except:
                            a- extracted DNA template
                            b- 2 specific primers
                            c- reation buffer
                            d- RNA polymerase

                            207- bacteriophage is :
                            a- virus that can be killed by antibiotic
                            b- virus that act like a bacteria
                            c- bacteria that act like a virus
                            d- virus that infect bacteria

                            208- all of the following viruses are transmitted by blood except:
                            a- HIV
                            b- HBV
                            c- HCV
                            d- herps virus

                            209-all of the following are RNA viruses except:
                            a- corona viridase
                            b- reoviridase
                            c- picorona viridase
                            d- pox viridase

                            210- penetration of naked virus is by :
                            a- fusion
                            b- endocytosis
                            c- translocation
                            d- all of the above

                            211- amniotic cavity inoculation is one type of virus culture in:
                            a- tissue wall
                            b- lab animals
                            c- embryonated egg
                            d- non of the above

                            212- …………….. is an invitro method amplification of a short sequence of target DNA
                            a- PCR
                            b- hyberdization
                            c- finger printing
                            d- all of the above

                            213- nucleic acid hyberdization means:
                            a- probe anneling or binding with it’s complementary
                            segment of NA
                            b- fragmentation of nucleic acid
                            c- amplification of nucleic acid
                            d- non of the above

                            214- DNA hyberdization is performed by:
                            a- primers
                            b- DNA labeled probe
                            c- restriction endonuclease
                            d- non of the above

                            215-PCR starts with :
                            a- annealing
                            b- denaturation of DNA
                            c-extension of primers
                            d- non of the above

                            216- ……………….. is a piece of DNA fragment of a particular gene that can bind specially with it’s complementary piece of DNA:
                            a- codon
                            b- probe
                            c- LCR
                            d- code

                            217- how many primers are used in PCR :
                            a- non
                            b- one
                            c- two
                            d- three

                            304-Lowenstein-Jensen media is used for the isolation for:
                            a- neisseria gonorrhea
                            b- mycobacterium tuberculosis
                            c- haemophilus influenza
                            d- staphylococcus aureus



                            305- Hekton-Enteric agar is the selective media for:
                            a- salmonella
                            b- streptococcus
                            c- staphylococcus
                            d- all of the above

                            306- the selective media for isolation of fungi is:
                            a- S-S agar
                            b- XLD agar
                            c- sabouraud glucose agar
                            d- Hekton-Enteric agar

                            307- gram positive rods with Chinese letter appearance:
                            a- mycobacterium tuberculosis
                            b- corynebacteria diphtheria
                            c- clostridium tetani
                            d- staphylococcus pneumonia

                            308- E lek test is done to diagnose :
                            a- streptococcus
                            b- staphylococcus aureus
                            c- clostridium tetani
                            d- corynebacteria diphtheria

                            309- all are lactose fermenter except:
                            a- E-COLI
                            b- proteus
                            c- klebsiella
                            d- enterobacter

                            310- produce pale colonies on MacConkey’s agar and have tendency to swarm on blood agar:
                            a- salmonella
                            b- shigella
                            c- klebsiella
                            d- proteus





                            311- lactose frementer gram negative bacilli with mucoid growth:
                            a- salmonella
                            b- shigella
                            c- klebsiella
                            d- proteus

                            312-non lactose fermenter gram negative bacilli produce H2S :
                            a- salmonella
                            b- shigella
                            c- klebsiella
                            d- proteus

                            313- Widal test is done for diagnosis of :
                            a- salmonella
                            b- shigella
                            c- E-coli
                            d- klebsiella

                            314- the causative agent of enteric fever:
                            a- salmonella
                            b- shigella
                            c- klebsiella
                            d- proteus

                            315- the most common cause of urinary tract infection :
                            a- E-coli
                            b- salmonella
                            c- shigella
                            d- streptococcus

                            316- an important cause of diarrhea in infant:
                            a- staphylococcus
                            b- E-coli
                            c- salmonella
                            d- shigella

                            317- the most common causative agent for peptic ulcer :
                            a- campylobacter
                            b- H-pylori
                            c- V-cholera
                            d- all of the above
                            318- TCBS is the selective media for isolation of:
                            a- H-pylori
                            b- V-cholera
                            c- E-coli
                            d- H influenza

                            319- gram negative bacilli strict aerobes grows on simple media producing a characteristic greenish pigment:
                            a- campylobacter
                            b- pseudomonas
                            c- pasterulla
                            d- bordetella

                            320- the bacteria which cause scarlet fever:
                            a- staphylococcus
                            b- streptococcus
                            c- salmonella
                            d- shigella

                            321- antistreptolysin O titer (ASO) is done for the diagnosis of:
                            a- group A streptococcus
                            b- group B streptococcus
                            c- staphylococcus aureus
                            d- staphylococcus albus

                            323- Loffler’s serum is used for isolation of:
                            a- anthrax
                            b- clostridium
                            c- diphtheria
                            d- T.B

                            324- gas gangrene is caused by:
                            a- clostridium tetani
                            b- clostridium botulinum
                            c- clostridium welchii
                            d- non of the above (( clostridium perfinges ))

                            325- Bacillary dysentery is caused by:
                            a- salmonella
                            b- shigella
                            c- cholera
                            d- all of the above

                            326- the cause of plague:
                            a- Y-enterocolitica
                            b- Y-pestis
                            c- Y pseudotuberculsois
                            d- non of the above

                            327- treponema palladium is the cause of :
                            a- T.B
                            b- gonorrhea
                            c- syphilis
                            d- AIDS

                            328- the venereal disease research laboratory test (VDRL) is done for diagnosis of:
                            a- T.B
                            b- gonorrhea
                            c- syphilis
                            d- AIDS

                            329- Trachoma is caused by:
                            a- mycoplasma
                            b- chlamydiae
                            c-richettsia
                            d- mycobacteria

                            391- M-tuberculsis bacilli stain with :
                            a- gram stain
                            b- Zheil Nelson stain
                            c- Gimesa stain
                            d- all of the above

                            392- AIDS is transmitted through :
                            a- food
                            b- blood
                            c- semen
                            d- (b) & (c)





                            393- bacteria which cause syphilis:
                            a- Neisseria gonorrhea
                            b- Viencent angina
                            c- Treponema palladium
                            d- Yersinia pestis

                            394- to diagnose syphilis:
                            a- RPR
                            b- VDRL
                            c- Wasserman
                            d- all of the above

                            395- streptococci secret:
                            a- streptolysin O
                            b- streptolysin S
                            c- streptokinase
                            d- all of the above

                            396- disease caused by streptococci:
                            a- scarlet fever
                            b- purperal sepsis
                            c- rheumatic fever
                            d- all of the above

                            397- staphylococci secrets:
                            a- coagulase enzyme
                            b- fibrinolysin
                            c- hyaluronidase
                            d- all of the above

                            398- gram positive bacilli:
                            a- Klebsilla
                            b- Salmonella
                            c- Proteus
                            d- C-diphtheria

                            399- meningeococcal meningitis is transmitted by :
                            a- food
                            b- droplet
                            c- touch
                            d- all of the above

                            400- dark field microscopy is used to diagnose :
                            a- T.B.
                            b- syphilis
                            c- gonorrhea
                            d- AIDS

                            401- it cause food poisining with flacid paralysis:
                            a- clostridium tetani
                            b- clostridium welchii
                            c- clostridium botulinium
                            d- all of the above

                            402- the infective stage of plasmodium vivax :
                            a-merozoites
                            b- sporozoites
                            c- schizont
                            d- trophozoite

                            403- Pirenella conica snail is the intermediate host of:
                            a- schistosoma haematobium
                            b- fasciola hiptica
                            c- heterphyes heterophyes
                            d- diphyllobothrium latum

                            404- Bulinus truncates snail is the intermediate host of :
                            a- fasciola hepatica
                            b- fasciola gigantica
                            c- schistosoma haematobium
                            d- schistosoma mansonii

                            405- to isolate meningiococci we have to culture the sample on:
                            a- Bordet Gengou
                            b- modified Thayer martin media
                            c- Lowenstein Jensen media
                            d- all of the above

                            406- to isolate fungi :
                            a- Brain-Heart infusion media
                            b- tissue culture
                            c- Lowenstein –Jensen media
                            d- chocolate agar

                            407- to isolate H- influenza:
                            a- blood agar
                            b- chocolate agar
                            c- mac Conkey media
                            d- all of the above

                            408- the bacteria which cause pseudomembrainous conjunctivitis :
                            a- N.gonorrhea
                            b- C. diphtheria
                            c- staphylococcus
                            d- Chlamydia

                            409- the best sample to diagnose meningitis :
                            a- blood
                            b- sputum
                            c- CSF
                            d- urine

                            410- used to stain Chlamydia
                            a- gram stain
                            b- giemsa stain
                            c- wright stain
                            d- all of the above
                            411-……….. is used as transport medium for sample in which cholera is suspected
                            a- Cary-Blair media
                            b- Stuart media
                            c- Alkaline peptone water
                            d- glycerol

                            412- the color of XLD medium:
                            a- green
                            b- red
                            c- yellow
                            d- blue

                            413- CIN medium is used to isolate:
                            a- E.coli
                            b- Vibrio cholera
                            c- yersinia
                            d- salmonella

                            414- to make wet mount preparation:
                            a- 10 % KOH
                            b- 10 % Na OH
                            c- 10 % Na CO3
                            d- 10% Na Cl

                            415- we do wet mount preparation for vaginal smear To diagnose:
                            a- T.vaginalis
                            b- N.gonorrhea
                            c- streptococci
                            d- staphylococci

                            416- to isolate viruses:
                            a- Loeffler media
                            b- tissue culture
                            c- Bordet –Gengou media
                            d- Brain- Heart infusion

                            417- to diagnose whooping cough :
                            a- Bordet –Gengou media
                            b- Lowenstein –Jensen media
                            c- modified Thayer martin media
                            d- New York city agar

                            418- to diagnose systemic infection we do :
                            a- urine culture
                            b- CSF culture
                            c- blood culture
                            d- sputum culture

                            419- we give no growth for blood culture after:
                            a- 1 week
                            b- 8 weeks
                            c- 6 weeks
                            d- 3 weeks





                            420- to dissolve mucous in sputum sample :
                            a- 10% NaOH
                            b- 30 % NaOH
                            c- 10 % KOH
                            d- 10% NaCl

                            421- mutualism means:
                            a- one partener benefits , other unaffected
                            b- both partner benefit
                            c- one partner benefit , other damaged
                            d- living together

                            422- Commensalisms means:
                            a- living together
                            b- one partner benefit , other damaged
                            c- both partner benefit
                            d- one partner benefits , other unaffected

                            423- Balantidium coli moves by:
                            a- flagella
                            b- cilia
                            d- pseudopod
                            d- all of the above

                            424- Mouth inhabitant:
                            a- Trichomonas hominis
                            b- Trichomonas tenax
                            c- Trichomonas vaginalis
                            d- giardia lamblia

                            425- Transmitted by sexual intercourse:
                            a- toxoplasma
                            b- giardia lamblia
                            c- Trichomonas vaginalis
                            d- all of the above

                            465- citrate utilization test is done to assist identification of:
                            a- gram +ve bacteria
                            b- gram –ve bacteria
                            c- entrobacteria
                            d- enterococcus

                            466- the Kovac's reagent used in the following biochemical tests:
                            a- catalase
                            b- coagulase
                            c- indole
                            d- methyl red

                            467- positive results for H2S production appear as …. Colour:
                            a- black
                            c- yellow
                            c- red
                            d- green

                            468-………. Give positive coagulase test:
                            a- streptococci
                            b- staphylococcus aureus
                            c- staphylococcus saprophyticus
                            d- Escherichia coli

                            469- methyl red test is performed with:
                            a- Erlich reagent
                            b- Kovac's reagent
                            c- Voges proskaur
                            d- non of the above

                            470- ………… give positive result with urease test:
                            a- salmonella
                            b- shigella
                            c- Y. enterocolitica
                            d- all of the above

                            471-…………. test is used to differentiate between bacteroides & brucella:
                            a- indole
                            b- methyl red
                            c- H2S production
                            d- nitrate reduction

                            472- DNAase test is positive with:
                            a- streptococcus pneumonia
                            b- E.coli
                            c- staphylococcus aureus
                            d- staphylococcus epidermis

                            473- ………… solution used in the gram stain technique acts as a mordant:
                            a- crystal violet
                            b- safranine
                            c- iodine
                            d- alcohol

                            474- the counter stain in Ziehl- Neelson stain is :
                            a- malachite green
                            b- methylene blue
                            c- iodine
                            d- (a) & (b)

                            475- bile solubility test is positive with :
                            a- streptococcus viridans
                            b- streptococcus pneumonia
                            c- streptococcus agalectiae
                            d- streptococcus pyrogenes

                            476-litmus milk decolorization test assist the identification of :
                            a- entrobacteria
                            b- bacteroides
                            c- brucella
                            d- enterococci










                            CHEMISTRY

                            330- ………….. are substance produced by specialized cells of the body and carried by blood stream where it affect other specialized cells:
                            a- vitamins
                            b- enzymes
                            c- isoenzyme
                            d- hormones

                            331- …………… is a protein which catalyse one or more specific biochemical reaction and not consumed during the reaction:
                            a- enzymes
                            b- hormones
                            c- vitamins
                            d- proteins

                            332- ………… is required in the hepatic synthesis of prothrombin and the blood clotting factors and it’s deficiency is observed in newborn infants:
                            a- vitamin E
                            b- vitamin A
                            c- vitamin K
                            d- vitamin D

                            333- there are enzymes that catalyze the same reaction but differ in there physical properties:
                            a- vitamins
                            b- adjuvents
                            c- isoenzyme
                            d- hormones

                            334- it’s functionis to maintain adequate serum level of calcium:
                            a- vitamin E
                            b- vitamin A
                            c- vitamin K
                            d- vitamin D




                            335-the inhibitor and substrate bind at different sites on the enzyme this type of inhibition is called:
                            a- competitive inhibition
                            b- non competitive inhibition
                            c- surface recognition
                            d- product concentration

                            336- ……….. found in cartilage consist of a core protein to which the linear carbohydrates chain are covalently attached:
                            a- glycoprotein
                            b- proteoglycan
                            c- link protein
                            d- hyaluronic acid

                            337- …………… is synthesized only by micro-organism , it’s not present in plants but present in liver , it’s deficiency leads to pernicious anemia:
                            a- vit C
                            b- vit B12
                            c-vit B1
                            d- vit B2

                            338- the brown color of the stool is due to the presence of:
                            a- urobilinogen
                            b- urobilin
                            c- porphyrin
                            d- bilirubin

                            339- the degradation of heme takes place in the …………. particularly in the liver and spleen
                            a- reticulocytes
                            b- erythrocytes
                            c- reticuloendothelial cells
                            d- non of the above

                            340- increased Hb destruction , the liver is unable to cup the greater load of pigment and bilirubin level well rises this is called :
                            a- hepatogenous jaundice
                            b- hemolytic jaundice
                            c- obstructive jaundice
                            d- non of the above

                            341- ………… plays a role in visual cycle it’s deficiency leads to night blindness, β-carotene is the major precursor of this vitamin in human:
                            a- vitamin E
                            b- vitamin A
                            c- vitamin K
                            d- vitamin D

                            342- a large percentage of the ……….. requirement in humans is supplied by intestinal bacteria:
                            a- biotin
                            b- niacin
                            c- folic acid
                            d- thiamin

                            343- ………….. are organic compounds required by the body in trace amount , can’t be synthesized by humans , but supplied by diet:
                            a- enzymes
                            b- vitamins
                            c- hormones
                            d- proteins

                            344- it facilitate the absorption of iron by reducing it to ferrus state in the stomach and it’s deficiency result in scurvy:
                            a- vit C
                            b- vit B
                            c- vit D
                            d- vit A

                            345- it’s function is to transport oxygen from the lung to the tissue:
                            a- haptoglobin
                            b- hemoglobin
                            c- bilirubin
                            d- myoglobin





                            346- ………….. act as an antioxidant and it’s deficiency cause liver degeneration:
                            a- vit E
                            b- vit A
                            c- vit K
                            d- vit D

                            347- it’s caused by liver parenchyma damage , the excretion of bile greatly decreased and the concentration of bilirubin in the blood rise :
                            a- hemolytic jaundice
                            b- hepatogenous jaundice
                            c- obstructive jaundice
                            d- non of the above

                            348- ……… play an essential role in body metabolism , a deficiency or excess may lead to serious dearrangement of body function:
                            a- enzymes
                            b- hormones
                            c- vitamins
                            d- isoenzymes

                            349- …………. will interfere with the chemical determination of bilirubin , giving high variable results:
                            a- hemolysis
                            b- hemoglobin
                            c- vitamins
                            d- hormones
                            350- in hemolytic jaundice there will be increase ……………. in serum:
                            a- direct bilirubin
                            b- indirect bilirubin
                            c- total bilirubin
                            d- all of the above




                            351- the inhibitor binds reversibly to the same site on the enzyme that the substrate normally occupy , this type of inhibition is called :
                            a- competitive inhibition
                            b- non competitive inhibition
                            c- surface recognition
                            d- product concentration

                            352- regulation of blood glucose level can be achieved by :
                            a- hormonal mechanism
                            b- hepatic and renal mechanism
                            c- (a) & (b)
                            d- non of the above

                            353- insulin is a hormone secretes by :
                            a- α cell of islet of langerhans in pancreas
                            b- β cell of islet of langerhans in pancreas
                            c- suprarenal cortex
                            d- non of the above

                            354- cholesterol is a component of all cell membrane and it’s the precursor of :
                            a- bile acid
                            b- steroid hormones
                            c- vit D
                            d- all of the above

                            355- anti diuretic hormone ( ADH) secretion is controlled by:
                            a- rennin angiotensin
                            b- plasma osmlality
                            c- (a) & (b)
                            d- non of the above

                            356- the electrophoresis is based on differential migration of :
                            a- charged particles
                            b- uncharged particles
                            c- molecular weight
                            d- (a) & (b)




                            357- acid base balance is regulated by :
                            a- oxygen concentration
                            b- hydrogen ion concentration
                            c- nitrogen ion concentration
                            d- (a) & (b)

                            358- it’s an increase in hydrogen ion concentration of the blood:
                            a- acidosis
                            b- alkalosis
                            c- acid base balance
                            d- (a) & (b)

                            359- over production of acid associated with :
                            a- diabetes mellitus
                            b- lactic acidosis
                            c- methanol poisoning
                            d- all of the above

                            360- serum bicarbonate is decreased in:
                            a- respiratory acidosis
                            b- metabolic acidosis
                            c- renal tubular acidosis
                            d- all of the above

                            361- chronic deficiency in dietry calcium can lead to :
                            a- anemia
                            b- bronchial asthma
                            c- osteoporosis
                            d- non of the above

                            362- ………… is due to decrease blood CO2:
                            a- metabolic acidosis
                            b- respiratory acidosis
                            c- respiratory alkalosis
                            d- metabolic acidosis

                            363- …………. is the most important factor affecting body sodium content:
                            a- aldosteron secretion
                            b- antidiuretic hormone
                            c- testosterone
                            d- all of the above
                            364- haemosiderosis is :
                            a- increase iron store
                            b- decrease iron store
                            c- increase hemoglobin
                            d- decrease hemoglobin

                            365- if there’s a mixture of protein ( colloids) and salt
                            ( crystalloid) they can be separated by :
                            a- precipitation
                            b- dialysis
                            c- chromatography
                            d- electrophoresis

                            366- the predominant cation in intracellular fluid is :
                            a- sodium
                            b- potassium
                            c- calcium
                            d- phosphorus

                            367- metabolic acidosis is due to :
                            a- failure to secret acid
                            b- bronchial asthma
                            c- loss of bicarbonate
                            d- (a) &(c)

                            368- high level of plasma ferritin may occur due to :
                            a- inflammatory condition
                            b- malignant disease
                            c- liver disease
                            d- all of the above

                            369- gonadal hormones estimation is important in :
                            a- detection of ovulation
                            b- assessment of amenorrhea
                            c- evaluation of delayed puberty
                            d- all of the above

                            370- the secretion of gonadal hormone is controlled by :
                            a- LH
                            b- FSH
                            c- TSH
                            d- (a) & (b)
                            371- the intensity of the color is directly proportional to the ………. of the analyte in the solution:
                            a- dilution
                            b- contamination
                            c- concentration
                            d- observation

                            372- the ………… contain information of any health or safety rich associated with use or exposure to hazardous chemicals:
                            a- MSDS
                            b- NFPA
                            c- POLT
                            d- OSHA

                            373- instraument used to measure color changes in the labs:
                            a- microscope
                            b- centrifuge
                            c- photometer
                            d- all of the above

                            374- the color coded signs used to identify flammable chemicals:
                            a- blue
                            b- yellow
                            c- white
                            d- red

                            375- quality assurance includes :
                            a- personal orientation
                            b- laboratory documentation
                            c- knowledge of laboratory istraumentation
                            d- all of the above

                            376- the laboratory procedure manual include:
                            a- patient preparation
                            b- specimen collection & processing
                            c- specimen preservation , storage & transport
                            d- all of the above





                            377- the principal of reflectance photometer
                            a- measure the amount of light that pass through the solution
                            b- measure the amount of light that the solution absorbs
                            c- (a) & (b)
                            d- non of the above

                            378- the blood cell counter include :
                            a- aperture impedence cell counter
                            b- Geiger counter
                            c- microscopes
                            d- all of the above

                            379- it is mession is to save lives , prevent injuries , and protect health of all workers in the lab. :
                            a- MSDS
                            b- NFPA
                            c- POLT
                            d- OSHA

                            380- ……………. requires 3 hours at 140 C ْ or 1 hour at 160 C ْ for complete sterilization
                            a- hot air oven
                            b- autoclave
                            c- filteration
                            d- all of the above

                            381- arterial blood samples are essential to do :
                            a- CBC
                            b- urea
                            c-blood glucose
                            d- blood gas analysis

                            382- vaccum tubes with green stopper contain:
                            a- EDTA
                            b- sodium citrate
                            c- heparin
                            c- no anticoagulant





                            383- serum separator tube is all of the following except:
                            a- contain gel that separate serum from cells during centrifugation
                            b- contain clot activator to speed clot formation
                            c- has red & black mottled top stopper
                            d- used for coagulation tests

                            384- for phlepotomy we use all of the following except:
                            a- the hypodermic needle & syringe
                            b- the vaccum tube system
                            c- the monolet lancets
                            d- the winged infusion set

                            385- lab. equipment should be cleaned and disinfected with :
                            a- hypochlorite
                            b- formaldehyde
                            c- glutaraldehyde
                            d- (b) & (c)

                            386- any blood split in the lab should be immediately swabbed with :
                            a- hypochlorite
                            b- alcohol
                            c- soap
                            d- water

                            387- CBC is performed using:
                            a- serum
                            b- well mixed EDTA whole blood
                            c- plasma
                            d- non of the above

                            388- which tube should be filled first in blood collection:
                            a- tubes with anticoagulant
                            b- tube without anticoagulant
                            c- tubes for blood culture
                            d- non of the above






                            389- the monojector is designed to be used with :
                            a- the monolet lancet
                            b- tenderlett
                            c- tenderfoot
                            d- non of the above


                            390- the site of choice for capillary puncture in newborns is :
                            a- the earlobe
                            b- middle finger
                            c- the big toe
                            d- the lateral medial planter heel surface
















                            PARASITOLOGY

                            426- Intermediate host of Trypansom:
                            a- triatoma megista
                            b- sand fly
                            c- tse tse fly
                            d- anopheles

                            427- The cause of chaga's disease:
                            a- trypanosoma gambiense
                            b- trypansoma rhodesiense
                            c- trypansoma cruzi
                            d- leishmania braziliense

                            428- The cause of sleeping sickness:
                            a- trypanosoma gambiense
                            b- trypanosoma cruzi
                            c- trypanosoma rhodesiense
                            d- (a) & (c)

                            429- Cause Kala- azar:
                            a- leishmania tropica
                            b- leishmania braziliense
                            c- leishmania donovani
                            d- leishmania mexicana

                            430-cause oriental sore:
                            a- plasmodium ovale
                            b- leishmania tropica
                            c- leishmania donovani
                            d- trypanosoma rhodesiense

                            431- Its trophozite is shaped like a pear , has the 2 nuclei that resembles eyes and 4 pairs of flagella that look like hair:
                            a- Trichomonas vaginalis
                            b- entameoba histolytica
                            c- giardia lamblia
                            d- endolimax nana




                            432- Pear shaped trophozite with 4 anterior flagella and a 5th forming the outer edge of a short undulating membrane:
                            a- Trichomonas hominis
                            b- entameoba histolytica
                            c- entameoba coli
                            d- endolimax nana

                            433- sometimes it cause metastatic infection which involve liver, lung, brain or other viscera:
                            a- giardia lamblia
                            b- Trichomonas vaginalis
                            c- entameoba histolytica
                            d- balantidium coli

                            434- Intestinal ciliate:
                            a- entameoba histolytica
                            b- entameoba coli
                            c- giardia lamblia
                            d- balantidium coli

                            435- Asexual multiplication of plasmodium vivax takes place in:
                            a- anopheles
                            b- sand fly
                            c- human
                            c- tse tse fly

                            436- Plasmodium falciparam is transmitted by :
                            a- triatoma megista
                            b- tse tse fly
                            c- anopheles
                            d- sand fly

                            437- Moves by pseudopods:
                            a- giardia lamblia
                            b- balantidium coli
                            c- entameoba histolytica
                            d- Trichomonas vaginalis

                            438- it's one of the round worms:
                            a- schistosoma mansoni
                            b- schistosoma haematobium
                            c- ascaris lumbricoides
                            d- fasciola hepatica

                            439- it's one of the tape worms:
                            a- ascaris lumbricoides
                            b- ancylostoma duodenal
                            c- trichuris tricura
                            d- taenia saginata

                            440- barrel shapped egg, yellow brown in color with a colorless protruding mucoid plug in each end:
                            a- egg of ascaris lumbricoides
                            b- egg of ancylostoma duodenal
                            c- egg of trichuris tricura
                            d- egg of taenia saginata

                            441- large oval egg ,pale yellow brown in color has a characteristic side spine & contain a fully developed
                            miracidium , the worm is:
                            a- S.mansoni
                            b- S. hematobium
                            c- A. duodenal
                            d- T. solium

                            442- large oval egg , pale yellow brown in color has an indistinct operculum and contains unsegmented ovum:
                            a- S. hematobium
                            b`
                            c- heterophyes heterophyes
                            d- taenia solium

                            443- round egg , embryo is surrounded by a thick brown radially striated wall , hooklets are present in the embryo:
                            a- S. hematobium
                            b- fasciola hepatica
                            c- A. duodenal
                            d- T. solium

                            444-oval colorless egg,flattened on one side & contains a larvae:
                            a- hymenelopis diminuta
                            b- dipylidium caninum
                            c- entrobius vermicularis
                            d- taenia saginata

                            445- the cause of malignant malaria:
                            a- plasmodium vivax
                            b- plasmodium ovale
                            c- plasmodium malaria
                            d- plasmodium falciparum

                            446- infection occur when infective filariform larvae penetrate the skin:
                            a- ascaris lumbricoides
                            b- ancylostoma duodenal
                            c- fasciola hepatica
                            d- heterophyes heterophyes

                            447- ……..lives in the liver and bile ducts of sheep and cattle:
                            a- stronyloides stercoralis
                            b- schistosoma mansoni
                            c- fasciola hepatica
                            d- ancylostoma duodenal

                            448-segment found in stool which is white & opaque & measures 20 mm long by 6mm wide with uterus that has a central stem and more than 13 side branches on each side…the worm is :
                            a- fasciola hepatica
                            b- trichuris trichuris
                            c- heterophyes heterophyes
                            d- taenia saginata

                            449- infection is by eating raw or under cooked fish:
                            a- fasciola hepatica
                            b- trichuris trichuris
                            c- heterophyes heterophyes
                            d- taenia solium

                            450- …… is transmitted by eating raw or under cooked beef:
                            a- heterophyes heterophyes
                            b- taenia saginata
                            c- schistosoma mansoni
                            d- ancylostoma duodenal









                            BODY FLUID

                            451- urine output < 400 ml/24 hours is :
                            a- polyuria
                            b- anuria
                            c- oligouria
                            d- non of the above

                            452- precipitation of urates takes place in:
                            a- acidic urine
                            b- alkaline urine
                            c- neutral urine
                            d- all of the above

                            453- it's one of the causes of persistently acidic urine:
                            a- urinary tract infection
                            b- phenylketonurea
                            c- excessive bicarbonate ingestion
                            d- excessive ingestion of soda

                            454- common cause of proteinuria:
                            a- alcoholism
                            b- fasting > 18 hours
                            c- diabetes mellitus
                            d- Bence- Jones proteins

                            455- dipstick detect acetoacetic acid & acetone which react with:
                            a- peroxides
                            b- nitroprusside
                            c- diazo compounds
                            d- indoxyl esters

                            456- in dipstick bilirubin reacts with :
                            a- nitroprusside
                            b- peroxides
                            c- indoxyl esters
                            d- diazo compounds




                            457- among the common cause of hematouria:
                            a- urogenital carcinoma
                            b- diabetes mellitus
                            c- heavy exercise
                            d- metabolic disorder

                            458- large number of hyaline cast indicated:
                            a- acute pyelonephritis
                            b- proliferative glomerulonephritis
                            c- heart failure
                            d- all of the above

                            459- red cell casts indicates:
                            a- acute pyelonephritis
                            b- proliferative glomerulonephritis
                            c- heart failure
                            d- all of the above

                            460- crystals which look like envelope :
                            a- triple phosphate
                            b- cystine
                            c- uric acid
                            d- calcium oxalate

                            461- Biuret test is done to determine:
                            a- glucose b- pentose
                            c- protein d- galactose

                            462- the 1st tube of synovial fluid is for:
                            a- hematology b- chemistry
                            c- microbiology d- microscopy

                            463- square plate like crystals with notched corners in synovial fluid indicate:
                            a- uric acid b- calcium pyrophosphate
                            c- cholesterol d- monosodium urate

                            464- abnormal forms in semen should not exceed:
                            a- 10 % b- 5 %
                            c- 25 % d- 50 %











                            Lab.Management

                            477-The process of getting things done through and with people operating in organized group toward a common goal is the
                            a- management
                            b- Organization
                            c- Planning
                            d- None of the above

                            478- Primary objectives in the planning are directed to
                            a- the laboratory as a whole
                            b- Increase the efficiency in the performance of the lab. test
                            c- Decrease the costs in the performance of the lab. test
                            d- All of the above

                            479- Forecasting needs for staff personnel means
                            a-Prediction in relation to the kind of technician and technologist who will be working in the lab.
                            b- Plan for the full utilization of efficient use of instrument
                            c- Plan for the full use of space in the lab.
                            d- None of the above

                            480-An organization
                            a- Is formed when 2 or more persons are brought together to achieve a common goal
                            b- Is closely related to planning
                            c- Involves structuring activities and functions within institutions to
                            achieve the goals and objects
                            d- all of the above

                            481- The real behavior and relationships of organization members usually differ from their planned behavior and relationships. It is
                            a- Formal organization
                            b- Informal organization
                            c- Space utilization
                            d- None of the above



                            482- The intra lab. System includes the following except
                            a- Calendar format
                            b- Histogram format
                            c- out of limits report sheet
                            d- Proficiency testing and computer program

                            483- The out of limits report form provides
                            a- Space for recording reagents changes
                            b- Control lot number changes
                            c- Serve as a general "dairy" of the test methodology
                            d- All of the above

                            484- Patient preparation, specimen collection and technical performance of lab. test are general categories of…………..
                            a- Planning
                            b- Utilization of space
                            c- Work flow
                            d- Quality control

                            485-floor book manual includes the following except
                            a- Test name
                            b- Sample fluid
                            c- Minimum volume
                            d- Proper procedures for collecting routine and special tests

                            486- Collection procedure manual involve
                            a- Blood collection from pediatric patients
                            b- Intensive care blood collection
                            c- Isolation techniques for lab. Personnel
                            d- All of the above

                            487-On the container and \or lab requisition
                            a- Patient's full name should be put
                            b- Hospital number should be put
                            c- Date of collection should be put
                            d- All of the above

                            488 Accuracy referred to the following except
                            a- Correctness and exactness of the test
                            b- Closeness of the test to the true value
                            c- True value determined by comparison to a standard
                            d- reproducibility

                            489- Regarding precision the following is true except
                            a- Reproducibility
                            b- Closeness of the test results to one another when using the same specimen
                            c- In the clinical lab it is expressed as (SD) and coefficient of variation
                            d- The capability of the method to detect a small amount of substance with some assurance

                            490- Reliability is
                            a- The ability of a method to measure only that substance being tested
                            b- The ability of the test method to maintain its accuracy despite of
                            extraneous circumstances
                            c- The ability of the method to maintain accuracy, precision and ruggedness
                            d- None of the above

                            491- …………….. This symbol in the flow chart means
                            a- Beginning process
                            b- Decision
                            c- Manual operation
                            d- Decision mod

                            492- This symbol in the flow chart means ( )
                            a-Beginning process
                            b- Decision
                            c- Direction flow
                            d- Document

                            493- Work load on which personnel requirements are usually based is influenced by
                            a- changes in volume
                            b- Test mix
                            c- Patient population
                            d- All of the above





                            494- The physical features of the lab. one of the measures of
                            a- forecasting of personnel needs
                            b- Assessment of space utilization
                            c- Time management
                            d- None of the above

                            495- If P (E) is the probability of E we may express this definition as
                            a- P (E) = m\N
                            b- P (E) = N\m
                            c- P (E) = m X N
                            d- None of the above

                            496- When a test indicates a positive status when the true status is negative it is called
                            a- positive test
                            b- False positive test
                            c- Negative test
                            d- False negative test

                            497- The specificity of a test
                            a- The probability of a positive test results or (presence of the symptoms) given the presence of the disease
                            b- The probability of a negative test results or (absence of the symptoms) given the absence of the disease
                            c- a and b
                            d-None of the above

                            498- The largest collection of entities for which we have an interest at a particular time is called
                            a- Population
                            b- Sample
                            c- Data
                            d-All of the above

                            499- A sample is
                            a- A part of a population
                            b- The whole population
                            c- Endless population
                            d- None of the above

                            500- If we have 100 students and they are ranked by age beginning with the 4th student, every tenth student is chosen (the student no. 4 then 14 and 24 and so on) this type of sample is called
                            a- Systemically selected sample
                            b- A stratified selected sample
                            c- Simple random sample
                            d- Cluster selected sample

                            501-A point estimate is
                            a- A single numerical value used to estimate the corresponding population parameter
                            b- Tow numerical values defining a range of values include the parameter being estimated
                            c- a and b
                            d- None of the above

                            502-A statistical inference is
                            a- A procedure by which we reach a conclusion about population based on the information obtained from the sample drawn from it
                            b- The cause behind estimation in the health science fields
                            c- Calculated data from the data of the sample that are approximation of the corresponding parameter
                            d- None of the above

                            503- Estimator is
                            a- the rule that tells us how to compute the single value which is called estimate
                            b- Two numerical values defining the range of values
                            c- a, b
                            d- None of the above

                            504 The table which shows the way in which values of the variables are distributed among the specified class interval is called
                            a- Relative frequency
                            b- Ordered array
                            c- Frequency table
                            c- None of the above


                            505- The following are the ages of 5 patients seen in the emergency room in certain day 35, 30, 55, 40, 25 years the mean of their ages is
                            a- 37 years
                            b- 30 years
                            c- 39 years
                            d- 40 years

                            500سؤال انشاء الله لم يخرج عنهم الامتحان واللى يريد الاجابات ممكن يراسلنى
                            والله أعلم
                            زاهر............. مصر ...............جده

                            تعليق


                            • #15
                              اخ زاهر الله يجزاك الف خير يارب

                              معليش انتا كيميا حيوي بكالوريس ؟؟

                              وهل اختبرت والاختبار نفس هالاسئله

                              ومتى اختبرت .؟؟

                              ياريت تفيدنا الله يجزاك الخير

                              تعليق

                              يعمل...
                              X